266 views
0 votes
0 votes
Let $G = (V,T,S,P)$ be a context-free grammar such that every one of its productions is of the form $A → v,$ with $|v| = k > 1.$ Show that the derivation tree for any $w ∈ L(G)$ has a height $h$ such that

                                           $\log_{k}|w|\leq h\leq \frac{(|w|-1)}{k-1}$.

Please log in or register to answer this question.

Related questions

0 votes
0 votes
0 answers
2
0 votes
0 votes
0 answers
3
0 votes
0 votes
0 answers
4